Themenbereiche Themenbereiche Profile Hilfe/Anleitungen Help    
Recent Posts Last 1|3|7 Days Suche Suche Tree Tree View  

a+b=Pi/4 <=> sin²(a)+sin(a)*cos(a)+si...

ZahlReich - Mathematik Hausaufgabenhilfe » ---- Archiv: Klassen 8-10 » Trigonometrie » Beweise » a+b=Pi/4 <=> sin²(a)+sin(a)*cos(a)+sin²(b)+sin(b)*cos(b) = 1 « Zurück Vor »

Autor Beitrag
Seitenanfangvoriger Beitragnächster BeitragSeitenende Link zu diesem Beitrag

Torben
Suche alle Beiträge dieser Person in dieser Hauptrubrik
Unregistrierter Gast
Veröffentlicht am Dienstag, den 20. August, 2002 - 11:17:   Beitrag drucken

Guten Tag!


Gegeben: Winkel a, b mit 0 < a+b < Pi.

Zeige, dass die Gleichung sin²(a)+sin(a)*cos(a)+sin²(b)+sin(b)*cos(b) = 1 dann äquivalent zu a+b=Pi/4 ist.
Seitenanfangvoriger Beitragnächster BeitragSeitenende Link zu diesem Beitrag

Torben
Suche alle Beiträge dieser Person in dieser Hauptrubrik
Unregistrierter Gast
Veröffentlicht am Dienstag, den 20. August, 2002 - 11:22:   Beitrag drucken

sorry, ich meinte:
gegeben: 0 < a,b < Pi
also 0 < a < Pi und 0 < b < Pi
Seitenanfangvoriger Beitragnächster BeitragSeitenende Link zu diesem Beitrag

egal
Suche alle Beiträge dieser Person in dieser Hauptrubrik
Unregistrierter Gast
Veröffentlicht am Dienstag, den 20. August, 2002 - 12:03:   Beitrag drucken

Hi Torben,

die Äquivalenz stimmt nicht. Die Gleichung ist äquivalent zu
(-cos(2a + p/4) - cos(2b + p/4))/√2 + 1 = 1
cos(2a + p/4) + cos(2b + p/4) = 0
also
2a + p/4 = 2b + p/4 + p + 2np für ganzzahliges n
a - b = p/2 + np
wegen 0 < a,b < p ist das äquivalent zu
|a-b|= p/2
Seitenanfangvoriger Beitragnächster BeitragSeitenende Link zu diesem Beitrag

Torben
Suche alle Beiträge dieser Person in dieser Hauptrubrik
Unregistrierter Gast
Veröffentlicht am Dienstag, den 20. August, 2002 - 13:52:   Beitrag drucken

Danke für die schnelle Antwort.
Aber a+b=Pi/4 scheint zu stimmen.
Jedenfalls eher als |a-b|= pi/2.

z.B. für a=Pi/6 und b=Pi/12 ist die Gleichung
sin²(a)+sin(a)*cos(a)+sin²(b)+sin(b)*cos(b) = 1 erfüllt, a+b=Pi/4 auch,
aber |a-b|= pi/2 nicht.


ich glaube, wegen cos(x)+cos(y)=0 <=> x+y=Pi
folgt aus der Gleichung
cos(2a + pi/4) + cos(2b + pi/4) = 0, dass
2a + Pi/4 + 2b + Pi/4 = Pi
<=> 2a+2b = Pi/2 ist,

aber durch welche Umformungen kommt man auf
(-cos(2a + pi/4) - cos(2b + pi/4))/&#8730;2 + 1 = 1 ?
Seitenanfangvoriger Beitragnächster BeitragSeitenende Link zu diesem Beitrag

egal
Suche alle Beiträge dieser Person in dieser Hauptrubrik
Unregistrierter Gast
Veröffentlicht am Dienstag, den 20. August, 2002 - 15:10:   Beitrag drucken

Hi Torben,

du hast mich missverstanden. Die von dir behauptete Äquivalenz ist falsch, weil a+b = p/4 nicht die einzige Lösung ist, es gibt die zweite Lösung |a-b| = p/2. Die Gleichung gilt z.B. auch für a=p/6 und b=2p/3.

Die Umformungen sind elementar. Bei sin²(a)+sin(a)*cos(a) zuerst sin(a) herausheben, dann die Formeln anwenden:
sin(x)+cos(x) = √2 sin(x+p/4)
sin(x)*sin(y) = (cos(x-y)-cos(x+y))/2
cos(-p/4) = 1/√2
Bei sin²(b)+sin(b)*cos(b) genauso.
Seitenanfangvoriger Beitragnächster BeitragSeitenende Link zu diesem Beitrag

Torben
Suche alle Beiträge dieser Person in dieser Hauptrubrik
Unregistrierter Gast
Veröffentlicht am Dienstag, den 20. August, 2002 - 17:11:   Beitrag drucken

Hallo nochmal
ich hatte noch Schwierigkeiten mit meinem Definitionsbereich der Winkel.

Wenn die Voraussetzung nun lautet:
0 < a < Pi und 0 < b < Pi und 0 < a+b < Pi,

gilt dann die Folgerung
sin²(a)+sin(a)*cos(a)+sin²(b)+sin(b)*cos(b) = 1
=> a+b=Pi/4 ?
Seitenanfangvoriger Beitragnächster BeitragSeitenende Link zu diesem Beitrag

egal
Suche alle Beiträge dieser Person in dieser Hauptrubrik
Unregistrierter Gast
Veröffentlicht am Dienstag, den 20. August, 2002 - 17:33:   Beitrag drucken

Nein, p/6 + 2p/3 ist kleiner als p
Seitenanfangvoriger Beitragnächster BeitragSeitenende Link zu diesem Beitrag

Torben
Suche alle Beiträge dieser Person in dieser Hauptrubrik
Unregistrierter Gast
Veröffentlicht am Dienstag, den 20. August, 2002 - 20:00:   Beitrag drucken

Danke für die Betreuung und die Geduld dabei.
Ich habe mir jetzt die Voraussetzungen nochmal überlegt:

gilt die Folgerung
sin²(a)+sin(a)*cos(a)+sin²(b)+sin(b)*cos(b) = 1
=> a+b=Pi/4 ,

wenn vorausgesetzt wird, dass 0<a<Pi/2 und 0<b<Pi/2 ist?
Seitenanfangvoriger Beitragnächster BeitragSeitenende Link zu diesem Beitrag

egal
Suche alle Beiträge dieser Person in dieser Hauptrubrik
Unregistrierter Gast
Veröffentlicht am Mittwoch, den 21. August, 2002 - 08:39:   Beitrag drucken

Hi Torben,

nichts zu danken, habs gern gemacht. Für 0 < a,b < p/2 ist die Lösung eindeutig.

Beitrag verfassen
Das Senden ist in diesem Themengebiet nicht unterstützt. Kontaktieren Sie den Diskussions-Moderator für weitere Informationen.

ad

Administration Administration Abmelden Abmelden   Previous Page Previous Page Next Page Next Page